Jump to content

vietfrog's Content

There have been 829 items by vietfrog (Search limited from 06-06-2020)



Sort by                Order  

#276172 Tản mạn BĐT

Posted by vietfrog on 12-09-2011 - 00:12 in Bất đẳng thức và cực trị

Thêm một BĐT nữa khá hay, mình đưa lên để mọi người cùng chém.

Bài 58:
Cho $a,b,c \ge 0$. Chứng minh rằng:

$({a^2} + 2)({b^2} + 2)({c^2} + 2) \ge 9(ab + ac + bc)$


P/s: Mọi người thử suy nghĩ trong trường hợp $a,b,c \in R$ nhé!



#277961 Tản mạn BĐT

Posted by vietfrog on 06-10-2011 - 16:29 in Bất đẳng thức và cực trị

Bài 66:Cho a,b,c là các số thực dương thỏa mãn abc=1.Chứng minh bất đẳng thức:
$
\dfrac{a}{a^{2}+3}+\dfrac{b}{b^{2}+3}+\dfrac{c}{c^{2}+3}\leq \dfrac{3}{4}$

Bài làm
Ta có: \[abc = 1 \Leftrightarrow \ln a + \ln b + \ln c = 0\]
Đặt :$\ln a = x;\ln b = y;\ln c = z \Rightarrow x + y + z = 0$
Khi đó: \[VT = \dfrac{{{e^x}}}{{{{({e^x})}^2} + 3}} + \dfrac{{{e^y}}}{{{{({e^y})}^2} + 3}} + \dfrac{{{e^z}}}{{{{({e^z})}^2} + 3}}\]

Xét : $f(x) = \dfrac{{{e^x}}}{{{{({e^x})}^2} + 3}}$( hàm lõm)




Theo BĐT tiếp tuyến ta có: $f(x) \le f'({x_0}).(x - {x_0}) + f({x_0})$
Với ${x_0} = 0$ ta có :\[f(x) \le f'(0).(x - 0) + f(0) = \dfrac{1}{8}x + \dfrac{1}{4}\]
Làm tương tự rồi cộng các BĐT cùng chiều ta được:

$f(x) + f(y) + f(z) \le \dfrac{1}{8}(x + y + z) + \dfrac{3}{4} = \dfrac{3}{4}$ (đpcm)





Dấu = xảy ra khi $x = y = z = 0 \Leftrightarrow a = b = c = 1$



#276029 Tản mạn BĐT

Posted by vietfrog on 11-09-2011 - 08:47 in Bất đẳng thức và cực trị

Bài 53
Xây dựng các bất đẳng thức tương tự ta phải chỉ phải chứng minh
$\dfrac{{{a^3} + {b^3} + {c^3} + 6}}{{{{\left( {{a^2} + {b^2} + {c^2}} \right)}^2}}} \le 1$
Nhưng rõ ràng bất đẳng thức này có thể dễ dàng cm bởi AM-GM vậy ta có đpcm

Bạn Hoàng chứng minh rõ chỗ này cho mọi người cũng xem đi.
Thanks!

P/s: Bài này vẫn còn 1 cách nữa.



#275641 Tản mạn BĐT

Posted by vietfrog on 08-09-2011 - 12:21 in Bất đẳng thức và cực trị

Giả sử x :( y =)) z
Theo Bất đẳng thức AM-GM ta có: $xy^{2}z^{3}\leq\dfrac{ (x+2y+3z)^{6} }{ 6^{6} }\leq\dfrac{ (2x+2y+2z)^{6} }{ 6^{6} }=\dfrac{ (x+y+z)^{6} }{ 3^{6} }$
=(( $\dfrac{ (x+y+z)^{6} }{xy^{2}z^{3} }\geq 3^{6}$
Vậy Min P=$ 3^{6}$ =)) x=y=z

Híc. Lời giải của bạn chưa đúng. Bạn thử thay :$x=1;y=2;z=3$ thì $P =432 <{3^6} = 729$
Bài này chỉ cần sử dụng BĐT AM-GM nhưng phân tách cho hợp lý là ok.
Mời mọi người tham gia giải tiếp.



#275615 Tản mạn BĐT

Posted by vietfrog on 08-09-2011 - 00:01 in Bất đẳng thức và cực trị


Bài 50:
Tìm Min của:
$P = \dfrac{{{{(x + y + z)}^6}}}{{x{y^2}{z^3}}}$
với x,y,z là các số dương



#283639 Tản mạn BĐT

Posted by vietfrog on 15-11-2011 - 23:30 in Bất đẳng thức và cực trị

Đây là 2 bài trong Thử sức trước kì thi của THTT

Bài 84:
Cho 2 số thực $x,y$ thỏa mãn: ${x^2} + y{}^2 = 4$
Tìm GTNN của:
\[P = \sqrt {5 - 2x} + \sqrt {54 - 2x - 14y} \]


Bài 85
Cho $x,y,z$ là các số thực dương thỏa mãn:$xy + yz + xz \le 3$
Chứng minh rằng:
\[\dfrac{2}{{\sqrt {xyz} }} + \dfrac{{27}}{{(2x + y)(2y + z)(2z + x)}} \ge 3\]



#284216 Tản mạn BĐT

Posted by vietfrog on 19-11-2011 - 22:57 in Bất đẳng thức và cực trị

Bài 89: Với A,B,C là 3 góc của 1 tam giác,chứng minh rằng:
$$3\sin{A}+4\sin{B}+5\sin{C}+5\sqrt{2(\sin^2{A}+\sin^2{B}+\sin^2{C})} \ge 8(\sin{A}+\sin{B}+\sin{C})$$

Bài này thấy kì kì sao ý. dark templar mà cho bài dễ vậy?
Theo Cauchy-Schwarz ta có:

\[\begin{array}{l}
5\sin A + 4\sin B + 3\sin C \le \sqrt {\left( {{{\sin }^2}A + {{\sin }^2}B + {{\sin }^2}C} \right)} .\sqrt {{5^2} + {4^2} + {3^2}} \\
\Leftrightarrow 5\sin A + 4\sin B + 3\sin C \le 5\sqrt {2\left( {{{\sin }^2}A + {{\sin }^2}B + {{\sin }^2}C} \right)} \\
\Leftrightarrow 3\sin A + 4\sin B + 5\sin C + 5\sqrt {2\left( {{{\sin }^2}A + {{\sin }^2}B + {{\sin }^2}C} \right)} \ge 8(\sin A + \sin B + \sin C)
\end{array}\]



#313923 Tản mạn BĐT

Posted by vietfrog on 02-05-2012 - 17:23 in Bất đẳng thức và cực trị

Bài 171.
Cho $a, b, c$ là các số thực dương thỏa mãn điều kiện $a+b+c=1$ . Chứng minh rằng :
$$a\sqrt{b}+b\sqrt{c}+c\sqrt{a}\le \dfrac{1}{\sqrt{3}}$$

Lời giải
Áp dụng BĐT Cauchy -Schwarz và AM-GM ta có:
\[a\sqrt b + b\sqrt c + c\sqrt a \le \sqrt {\left( {a + b + c} \right)\left( {ab + ac + bc} \right)} = \sqrt {ab + ac + bc} \le \sqrt {\frac{{{{(a + b + c)}^2}}}{3}} = \frac{1}{{\sqrt 3 }}\]
Dấu $=$ khi $a=b=c=1/3$



#313915 Tản mạn BĐT

Posted by vietfrog on 02-05-2012 - 16:57 in Bất đẳng thức và cực trị

Bài 170.
Cho $a, b, c$ là các số thực dương thỏa mãn $abc=1$ . Chứng ,minh rằng :
$$\dfrac{1+ab^2}{c^3}+\dfrac{1+bc^2}{a^3}+\dfrac{1+ca^2}{b^3}\ge \dfrac{18}{a^3+b^3+c^3}$$

Lời giải
Áp dụng lần lượt BĐT Cauchy -Schwarz , Minkowsky ,AM-GM, Giả thiết ta có: :D
\[\begin{array}{*{20}{l}}
{\frac{{1 + a{b^2}}}{{{c^3}}} + \frac{{1 + b{c^2}}}{{{a^3}}} + \frac{{1 + c{a^2}}}{{{b^3}}} \ge \frac{{{{\left( {\sqrt {1 + a{b^2}} + \sqrt {1 + b{c^2}} + \sqrt {1 + c{a^2}} } \right)}^2}}}{{{a^3} + {b^3} + {c^3}}} \ge \frac{{{{\left( {\sqrt {{{\left( {1 + 1 + 1} \right)}^2} + {{\left( {b\sqrt a + c\sqrt b + a\sqrt c } \right)}^2}} } \right)}^2}}}{{{a^3} + {b^3} + {c^3}}}}\\
{ \ge \frac{{{3^2} + {{\left( {3\sqrt[3]{{abc\sqrt {abc} }}} \right)}^2}}}{{{a^3} + {b^3} + {c^3}}} = \frac{{{3^2} + {3^2}}}{{{a^3} + {b^3} + {c^3}}} = \frac{{18}}{{{a^3} + {b^3} + {c^3}}}}
\end{array}\]



#299134 Tản mạn BĐT

Posted by vietfrog on 12-02-2012 - 20:56 in Bất đẳng thức và cực trị

Bài 130. Cho $x, y, z$ là các số thực dương thoả mãn điều kiện $xyz = 1$. Chứng minh rằng :
$$\dfrac{x^9 + y^9}{x^6 + x^3y^3 + y^6} + \dfrac{y^9 + z^9}{y^6 + y^3z^3 + z^6} + \dfrac{z^9 + x^9}{x^6 + z^3x^3 + x^6} \ge 2$$

Đặt ${x^3} = a,{y^3} = b,{z^3} = c$ suy ra $abc=1$
BĐT tương đương:$\sum {\frac{{{a^3} + {b^3}}}{{{a^2} + ab + {b^2}}}} \ge 2$
Giả sử:
$$\frac{{{a^3} + {b^3}}}{{{a^2} + ab + {b^2}}} \ge \frac{1}{3}\left( {a + b} \right) \Leftrightarrow {\left( {a - b} \right)^2}\left( {a + b} \right) \ge 0$$
Do $a,b >0$ nên giả sử đúng!
Suy ra:
$$\sum {\frac{{{a^3} + {b^3}}}{{{a^2} + ab + {b^2}}}} \ge \sum {\left( {\frac{1}{3}\left( {a + b} \right)} \right)} = \frac{2}{3}\left( {a + b + c} \right) \ge \frac{2}{3}.3\sqrt[3]{{abc}} = 2$$

P/s: Huy và Kiên quản lý hộ anh Topic này nhé. Bài nào chưa ai làm có thể bôi đỏ số bài cho dễ nhìn. :D. Cảm ơn 2 em!



#299130 Tản mạn BĐT

Posted by vietfrog on 12-02-2012 - 20:46 in Bất đẳng thức và cực trị

Mình thêm ít bài nữa để có nhiều lựa chọn hơn cho các bạn
Bài 129.Cho $a, b, c$ là các số thực dương thoả mãn điều kiện $a^2 + b^2 + c^2 = 3$ . Chứng minh rằng :
$$\dfrac{1}{1 + ab} + \dfrac{1}{1 + bc} + \dfrac{1}{1 + ca} \ge \dfrac{3}{2}$$

Sử dụng BĐT AM-GM ta có được :

$$\sum {\frac{1}{{1 + ab}}} \ge \sum {\frac{1}{{1 + \frac{{{a^2} + {b^2}}}{2}}}} \ge \frac{9}{{\sum {1 + \frac{{{a^2} + {b^2}}}{2}} }} = \frac{9}{{3 + {a^2} + {b^2} + {c^2}}} = \frac{9}{6} = \frac{3}{2}$$



#275024 Tản mạn BĐT

Posted by vietfrog on 03-09-2011 - 12:18 in Bất đẳng thức và cực trị

Chúng ta sẽ cùng nhau thảo luận về 2 bài sau:

Bài 47 (Một BĐT nhẹ nhàng)
Cho $a;b;c;x;y;z \ge 0$ thỏa mãn: $a + x = b + y = c + z$
Chứng minh rằng:
$ax + by + cz \le 1$


Bài 48: Với a,b,c là các số dương. Chứng minh rằng:
$\dfrac{(b+c-a)^2}{(b+c)^2+a^2}+\dfrac{(a+c-b)^2}{(a+c)^2+b^2}+\dfrac{(a+b-c)^2}{(a+b)^2+c^2} \geq \dfrac{3}{5}$
P/s: Chỉ dùng Cauchy.


@vietfrog: bboy114crew có cách giải Bài 48 thì post lên cho mọi người tham khảo nhé!

P/s: Topic dạo này trầm quá. Để chào mừng diễn đàn mới mọi người hãy tham gia giải bài và post bài nhiều nhé :geq!



#273869 Tản mạn BĐT

Posted by vietfrog on 24-08-2011 - 20:35 in Bất đẳng thức và cực trị

Chúng ta đã có 6 lời giải cho BĐT AM-GM 3 số. Các bạn có cách giải khác thì cứ tiếp tục post ( Nên đúng cấu trúc cây nhé )
Tiếp đến chúng ta sẽ tìm đến BĐT khá nhiều ứng dụng:
-----BĐT Schur với 3 số với cách diễn đạt sau:
Bài 43
Cho $a,b,c \ge 0$. Chứng minh rằng:

${a^3} + {b^3} + {c^3} + 3abc \ge ab(a + b) + ac(a + c) + bc(b + c)$



P/s: Mọi người post thật nhiều cách hay lên nhé!



#268061 Tản mạn BĐT

Posted by vietfrog on 10-07-2011 - 21:10 in Bất đẳng thức và cực trị

Dấu = xảy ra khi $n=1$
Với $n\ge 2$ thì dấu bằng ko thể xảy ra do $\dfrac{1}{2}\ne \dfrac{1}{3} $ (AM-GM)

Rất cảm ơn ý kiến của hoangduc. Bạn có thể post vài bài BĐT lên để mọi người cùng làm được không?
Thanks!



#268561 Tản mạn BĐT

Posted by vietfrog on 15-07-2011 - 07:48 in Bất đẳng thức và cực trị

Xin phép làm trước :Leftrightarrow

$P=ba+cb+ac=(b-c)a+(c-a)(a+b)+a(a+b+c)$
$\ge 3(b-c)+6(c-a)+12a=3b+3c+6a=3(a+b+c)+3a\ge 45 $

Đẳng thức xảy ra khi $a=c=3, b=6 $

Thành thật phê bình bạn hoangduc. x-(
Lời giải bạn hay như thế sao bạn không post thêm vài bài BDT để mọi người để mọi người cùng làm.
Tôi đề nghị bạn post ngay 1 bài cho tôi. :x :x :Leftrightarrow



#267825 Tản mạn BĐT

Posted by vietfrog on 07-07-2011 - 21:09 in Bất đẳng thức và cực trị

Tú không post bài thì để anh tiếp chiêu vậy.

Bài 9:Chứng minh rằng: $\forall n$ nguyên dương ta có:

$\sqrt[n]{{(n + 1)!}} \ge 1 + \sqrt[n]{{n!}}$
p/s:Có ai chém không?



#267079 Tản mạn BĐT

Posted by vietfrog on 30-06-2011 - 20:21 in Bất đẳng thức và cực trị

Bài 1 :
$ S = \sum \dfrac{ab}{a+b+2c} =\sum \dfrac{ab}{(a+b)+(b+c)} \leq \dfrac{1}{4}( \sum ( \dfrac{ab}{a+c}+\dfrac{ab}{b+c}) =\dfrac{3}{4} $

Lâm có thể giải chi tiết hơn một chút cho mọi người dễ quan sát được không? Hộ mình nhé!



#267078 Tản mạn BĐT

Posted by vietfrog on 30-06-2011 - 20:13 in Bất đẳng thức và cực trị

Em xin trình bày một cách cho bài 2, đơn giản dễ hiểu :
Ta có :
$S=\dfrac{{ab}}{{2 - c}} + \dfrac{{ac}}{{2 - b}} + \dfrac{{bc}}{{2 - a}}=\dfrac{ab}{a+b}+\dfrac{ac}{a+c}+\dfrac{bc}{b+c}$
Do : $(a+b)^2 \ge 4ab \Leftrightarrow \dfrac{ab}{a+b} \le 4(a+b)$
Chứng minh tương tự ta có:$\dfrac{bc}{b+c} \le 4(b+c); \dfrac{ca}{c+a} \le 4(a+c)$
Vậy $S=\dfrac{ab}{a+b}+\dfrac{ac}{a+c}+\dfrac{bc}{b+c} \le 8(a+b+c)=16$
MaxS=16 khi và chỉ khi $a=b=c=\dfrac{2}{3}$

Tú làm nhanh ghê. Nhưng nên cẩn thận nhé. Tú nhầm chỗ này :
${(a + b)^2} \ge 4ab \Leftrightarrow \dfrac{{ab}}{{a + b}} \le \dfrac{{a + b}}{4}$
Vì vậy Max bằng 1.Hi



#269940 Tản mạn BĐT

Posted by vietfrog on 27-07-2011 - 20:04 in Bất đẳng thức và cực trị

Ta có bất đẳng thức thứ nhất tương đương với
$a + b + c + ab + bc + ca \ge 3\left( {\sqrt[3]{{abc}} + \sqrt[3]{{{a^2}{b^2}{c^2}}}} \right)$
Nó đúng bởi AM GM
Bất đẳng thức tổng quát cần sử dụng AM GM tinh tế hơn các bạn thử suy nghĩ vì suy cho cùng nó cũng không quá khó có thể tìm được dựa vào cách cm BCS bởi AM GM

Một cách nữa cho bài 1. ( để Chứng minh bài tổng quát)
Ta có Bất đẳng thức tương đương:
$1 + \sqrt[3]{{\prod a }} \le \sqrt[3]{{\prod {(1 + a)} }} \Leftrightarrow \sqrt[3]{{\dfrac{{1.1.1}}{{\prod {(1 + a)} }}}} + \sqrt[3]{{\dfrac{{\prod a }}{{\prod {(1 + a)} }}}} \le 1$
Áp dụng AM-GM có ngay:
$\sqrt[3]{{\dfrac{{1.1.1}}{{\prod {(1 + a)} }}}} + \sqrt[3]{{\dfrac{{\prod a }}{{\prod {(1 + a)} }}}} \le \dfrac{1}{3}\left[ {\sum {\dfrac{1}{{a + 1}} + \sum {\dfrac{a}{{a + 1}}} } } \right] = 1$



#269949 Tản mạn BĐT

Posted by vietfrog on 27-07-2011 - 21:09 in Bất đẳng thức và cực trị

Thêm một Bất đẳng thức nữa khá hay:
Đây là một câu trong đề thi HSG tỉnh Hải Dương 2008-2009:
Bài 21:Cho $x,y,z$ là các số thực dương thỏa mãn :$\left\{ \begin{array}{l} x + y + z = 9 \\ x \ge 5;x + y \ge 8 \\ \end{array} \right.$
Chứng minh rằng : $xyz \le 15$
(sơ sơ bài này cũng có 3 cách )



#271045 Tản mạn BĐT

Posted by vietfrog on 06-08-2011 - 11:08 in Bất đẳng thức và cực trị

Lâu lém mới lên chém bài này!
Theo dụng BĐT AM-GM cho n số dương ta có:
$\dfrac{(a+b)(n-1)}{c} + 1+...+1 \geq n\sqrt[n]{\dfrac{(a+b)(n-1)}{c} }$
( n-1 số 1)
$\Leftrightarrow \dfrac{(n-1)(a+b+c)}{nc} \geq \sqrt[n]{\dfrac{(a+b)(n-1)}{c} }$
$\Leftrightarrow \dfrac{n}{n-1}\sqrt[n]{n-1}.\dfrac{a}{b+c} \leq \sqrt[n]{\dfrac{c}{b+a}}$
xây dựng tương tự hai BDT rồi cộng lại ta được:
$\sqrt[n]{\dfrac{a}{b+c}}+\sqrt[n]{\dfrac{b}{c+a}}+\sqrt[n]{\dfrac{c}{a+b}} \geq \dfrac{n}{n-1}\sqrt[n]{n-1}$
Dấu = xảy ra khi :
$n=\dfrac{3}{2}$
a=b=c
p\s: topic này hay ghê!

Đề của mình : n là số tự nhiên, nên lời giải của mình không xảy ra dấu =. Cách giải này mình đã post ở trên rồi mà :D:D:D



#270836 Tản mạn BĐT

Posted by vietfrog on 04-08-2011 - 22:57 in Bất đẳng thức và cực trị

Nói chung em làm như vậy là cũng ổn rồi. May là anh bỏ dấu ''='' đi chứ không em lại tưởng sai đề. Bài giải của em có lẽ giống với gợi ý của anh dark templar:

$\left(\dfrac{a}{b+c} \right)^{k}+\left(\dfrac{b}{a+c} \right)^{k}+\left(\dfrac{c}{a+b} \right)^{k} \ge \min \left\{2;\dfrac{3}{2^{k}} \right\}$

Xin trình bày lời giải bài toán tổng quát:

Áp dụng AM-GM cho n số:

$\dfrac{{(a + b)(n - 1)}}{c},1,1,..,1$

Ta có:
$\dfrac{{\dfrac{{(a + b)(n - 1)}}{c} + (n - 1)}}{n} \ge \sqrt[n]{{\dfrac{{(a + b)(n - 1)}}{c}}}$

$ \Leftrightarrow \dfrac{{(n - 1)(a + b + c)}}{{nc}} \ge \sqrt[n]{{\dfrac{{(a + b)(n - 1)}}{c}}}$

Nghịch đảo lên ta được:

$ \Leftrightarrow \dfrac{{nc}}{{(n - 1)(a + b + c)}} \le \sqrt[n]{{\dfrac{c}{{(a + b)(n - 1)}}}}$

$\sqrt[n]{{\dfrac{c}{{a + b}}}} \ge \dfrac{n}{{n - 1}}.\sqrt[n]{{n - 1}}.\dfrac{c}{{a + b + c}}$
Cộng lại ta có:

$\sum {\sqrt[n]{{\dfrac{a}{{b + c}}}} \ge } \dfrac{n}{{n - 1}}.\sqrt[n]{{n - 1}}.\sum {\dfrac{a}{{a + b + c}}} = \dfrac{n}{{n - 1}}.\sqrt[n]{{n - 1}}$
Giải điều kiện dấu = ta thấy dấu = không xảy ra.

Vậy
$\sqrt[n]{{\dfrac{a}{{b + c}}}} + \sqrt[n]{{\dfrac{b}{{c + a}}}} + \sqrt[n]{{\dfrac{c}{{a + b}}}} > \dfrac{n}{{n - 1}}.\sqrt[n]{{n - 1}}$



#270382 Tản mạn BĐT

Posted by vietfrog on 31-07-2011 - 22:02 in Bất đẳng thức và cực trị

Bài tiếp theo nhé. Mọi người làm xong nhớ post thêm một bài Bất đẳng thức khác.
Nghiêm cấm việc rũ bỏ trách nhiệm, làm xong thì bỏ đi :D :D
Cảnh cáo '' dark_templar '' nhớ :D :D
Bài tiếp
Bài 26:Cho $x,y>0$
thỏa mãn$x+y=1$
Tìm Min của :
$P = \dfrac{{{x^3} + {y^3} + x + y + 3xy}}{{{x^3} + {y^3}}} + \dfrac{2}{{xy.{{(x + y)}^2}}}$

P/s: Tìm dấu ''='' nhé. Không quên post thêm bài :D



#270319 Tản mạn BĐT

Posted by vietfrog on 31-07-2011 - 11:12 in Bất đẳng thức và cực trị

Cảm ơn bạn! Cách này khá hay nhưng việc dự đoán điểm rơi có đơn giản không bạn.

Việc dự đoán này cũng tương đối. Nhưng muốn có một lời giải đẹp thì kì công 1 chút cũng không sao mà. :D



#267064 Tản mạn BĐT

Posted by vietfrog on 30-06-2011 - 19:04 in Bất đẳng thức và cực trị

TẢN MẠN BẤT ĐẲNG THỨC

Topic với những Bất đẳng thức không quá khó, cách giải không quá cầu kì.

Mong rằng mọi người sẽ cùng trao đổi và thư giãn với Bất đẳng thức



Bài 1: ( Nhiều cách càng tốt)
Cho $a,b,c$ là các số dương thỏa mãn: $a+b+c=1$
Tìm Max :
$S = \dfrac{{ab}}{{1 + c}} + \dfrac{{ac}}{{1 + b}} + \dfrac{{bc}}{{1 + a}}$
Bài 2:(nhiều cách hơn nữa)
Cho $a,b,c$ dương thỏa: $a+b+c=2$
Tìm Max:
$S = \dfrac{{ab}}{{2 - c}} + \dfrac{{ac}}{{2 - b}} + \dfrac{{bc}}{{2 - a}}$
Bài 3:(tùy tâm)
CMR: với $n>2$ thì :
$C_n^0C_n^1C_n^2...C_n^n \le {\left( {\dfrac{{{2^n} - 2}}{{n - 1}}} \right)^{n - 1}}$